LSAT and Law School Admissions Forum

Get expert LSAT preparation and law school admissions advice from PowerScore Test Preparation.

User avatar
 Dave Killoran
PowerScore Staff
  • PowerScore Staff
  • Posts: 5853
  • Joined: Mar 25, 2011
|
#41288
Complete Question Explanation
(The complete setup for this game can be found here: lsat/viewtopic.php?t=15690)

The correct answer choice is (A)

When H delivers the fifth speech, then from the third rule we know that S must deliver the last speech. This leaves J, R, and T to occupy the second, third, and fourth slots. Because J and T cannot give consecutive speeches, they must be separated by R, who must give the third speech:
J95_Game_#1_#4_diagram 1.png
Accordingly, answer choice (A) is correct.
You do not have the required permissions to view the files attached to this post.
 jessamynlockard
  • Posts: 42
  • Joined: Jan 15, 2018
|
#44497
Hi,
I'm struggling with the inferences required for question #4:

"If K delivers the first speech and H delivers the fifth speech, which one of the following must be true."
User avatar
 Jonathan Evans
PowerScore Staff
  • PowerScore Staff
  • Posts: 726
  • Joined: Jun 09, 2016
|
#44516
Hi, Jessamyn,

Let's take a look at the setup for this question:

Image

In red, notice that we've indicated our new SRK block clue above the diagram. We've also connected the H — S sequencing clue to our block. We have now accounted for four of our six variables. Which variables have we not accounted for? T and J.

Let's discuss the implications of this H — SRK mega clue.
  1. T cannot come immediately before S because that would put R, S, and T in a row.
  2. T and J can't be next to each other. We have to keep an eye out for that.
  3. J can't come first or last.
Put it all together, and we have a very restrictive situation.

Let's start by thinking where the SRK block could go.
  • We know H is before S, so that means S cannot be in 1.
  • Could S be in 2? What would happen? If S were in 2, H would be in 1. Spaces 1, 2, 3, and 4 would be filled. What would be left? Spaces 5 and 6. Who is available to go in 5 and 6? Only T and J. The problem is they cannot be next to each other, so this doesn't work.
Thus far we have established that S cannot go in 1 or 2. Where could S go? All that's left are spaces 3 and 4. Let's see what happens. I have illustrated both scenarios above.
  • If S is in 3, then spaces 3, 4, and 5 are filled. Which variable do we have to worry about now? We need to think about J.
    Remember, J cannot go first or last. The only spaces that are left are 1, 2, and 6. Therefore J must be in 2. We are left with spaces 1 and 6. Since H is before S, H is in 1. Therefore, T must be in 6.
  • If S is in 4, then spaces 4, 5, and 6 are filled. What do we worry about now? We have to make sure of two things: (1) J and T are not next to each other, and (2) J cannot be first. Therefore we put H between J and T. We place T first and J in 3.
Notice now that we've illustrated the only two possible solutions that work for this question. What could be true?

D) K's speech is immediately before T's speech.

We saw this in the first of our two possibilities illustrated above.

I hope this helps!
 jessamynlockard
  • Posts: 42
  • Joined: Jan 15, 2018
|
#44530
Thanks so much!

Get the most out of your LSAT Prep Plus subscription.

Analyze and track your performance with our Testing and Analytics Package.